It's Just Roots

Algebra Level 2

The roots of the equation x 4 14 x 3 + 51 x 2 + a x + b = 0 x^4-14x^3+51x^2+ax+b=0 are found to be in an arithmetic progression . Find the value of a + b a+b .

-94 45 98 100

This section requires Javascript.
You are seeing this because something didn't load right. We suggest you, (a) try refreshing the page, (b) enabling javascript if it is disabled on your browser and, finally, (c) loading the non-javascript version of this page . We're sorry about the hassle.

3 solutions

Let the first term and the common difference of the arithmetic progression be a 1 a_1 and d d respectively. Then the four roots are a 1 \\ a_1 , a 1 + d a_1+d , a 1 + 2 d a_1+2d , and a 1 + 3 d a_1+3d . By Vieta's formula we have:

a 1 + ( a 1 + d ) + ( a 1 + 2 d ) + ( a 1 + 3 d ) = 4 a 1 + 6 d = 14 2 a 1 + 3 d = 7 . . . ( 1 ) a_1 + (a_1 + d) + (a_1 + 2d) + (a_1+3d) = 4a_1+6d = 14 \implies 2a_1 + 3d = 7 \quad ...(1)

a 1 ( a 1 + d ) + a 1 ( a 1 + 2 d ) + a 1 ( a 1 + 3 d ) + ( a 1 + d ) ( a 1 + 2 d ) + ( a 1 + d ) ( a 1 + 3 d ) + ( a 1 + 2 d ) ( a 1 + 3 d ) = 6 a 1 2 + 18 a d 1 + 11 d 2 = 51 . . . ( 2 ) a_1(a_1+d) + a_1(a_1+2d) + a_1(a_1+3d) + (a_1+d)(a_1+2d) + (a_1+d)(a_1+3d) + (a_1+2d)(a_1+3d) \\ = 6a_1^2 + 18ad_1 + 11d^2 = 51 \quad ...(2)

( 1 ) 2 : 4 a 1 2 + 12 a 1 d + 9 d 2 = 49 . . . ( 3 ) ( 2 ) ( 3 ) : 2 a 1 2 + 6 a 1 d + 2 d 2 = 2 . . . ( 4 ) ( 4 ) × 2 : 4 a 1 2 + 12 a 1 d + 4 d 2 = 4 . . . ( 5 ) ( 3 ) ( 5 ) : 5 d 2 = 45 d = 3 ( 1 ) : a 1 = 7 3 ( 3 ) 2 = 1 \begin{aligned} (1)^2: \ \ \ 4a_1^2 + 12a_1d + 9d^2 & = 49 & ...(3) \\ (2) - (3): \ \ \ \ \ 2a_1^2 + 6a_1d + 2d^2 & = 2 & ...(4) \\ (4) \times 2: \ \ \ 4a_1^2 + 12a_1d + 4d^2 & = 4 & ...(5) \\ (3)-(5): \ \ \ \ \ \qquad \qquad \qquad 5 d^2 & = 45 \\ \implies d & = 3 \\ (1): \ \ \ \ \ \qquad \qquad \implies a_1 & = \frac {7-3(3)}2 = -1 \end{aligned}

Then the four roots are 1 -1 , 2 2 , 5 5 , and 8 8 . And a a and b b are:

a = ( ( 1 ) ( 2 ) ( 5 ) + ( 1 ) ( 5 ) ( 8 ) + ( 1 ) ( 8 ) ( 2 ) + ( 2 ) ( 5 ) ( 8 ) ) = 14 b = ( 1 ) ( 2 ) ( 5 ) ( 8 ) = 80 a + b = 94 \begin{aligned} a & = - ((-1)(2)(5)+(-1)(5)(8)+(-1)(8)(2) + (2)(5)(8)) = -14 \\ b & = (-1)(2)(5)(8) = - 80 \\ \implies a+b & = \boxed{-94} \end{aligned}

Pi Han Goh
Dec 6, 2020

Let the roots of this quartic equation be A 3 D 2 , A D 2 , A + D 2 , A + 3 D 2 A - \frac {3D}2, A - \frac D2 , A + \frac D2, A + \frac {3D}2 , where D > 0 D> 0 .

By Vieta's formula , ( A 3 D 2 ) + ( A D 2 ) + ( A + D 2 ) + ( A + 3 D 2 ) = 14 A = 7 2 \left( A - \frac {3D}2\right) + \left( A - \frac D2 \right) + \left( A + \frac D2 \right) + \left(A+ \frac {3D}2 \right) = 14 \quad \implies \quad A = \frac 72 And the 2 nd 2^\text{nd} symmetric sum of these 4 numbers is equal to 51. For simplicities sake, let A 1 , A 2 , A 3 , A 4 A_1, A_2, A_3, A_4 be these numbers. A 1 A 2 + A 1 A 3 + A 1 A 4 + A 2 A 3 + A 2 A 4 + A 3 A 4 = 51 A_1 A_2 + A_1 A_3 + A_1 A_4 + A_2 A_3 + A_2 A_4 + A_3 A_4 = 51 Grouping them as such: A 1 ( A 2 + A 3 ) + A 4 ( A 2 + A 3 ) + A 1 A 4 + A 2 A 3 = ( A 1 + A 4 ) ( A 2 + A 3 ) + A 1 A 4 + A 2 A 3 A_1 (A_2 + A_3) + A_4 (A_2 +A_3) + A_1 A_4 + A_2 A_3 = (A_1 + A_4)(A_2 + A_3) + A_1 A_4 + A_2 A_3 Then substitute: 2 A 2 A + ( A 2 9 D 2 4 ) + ( A 2 D 2 4 ) = 51 2A \cdot 2A + \left( A^2 - \frac{9D^2}{4} \right) + \left( A^2 - \frac {D^2}4 \right) = 51 with A = 7 2 A = \frac72 , we get D = 3 D = 3 .

So the 4 roots are 1 , 2 , 5 , 8 -1,2,5,8 , thus the quartic equation is ( x + 1 ) ( x 2 ) ( x 5 ) ( x 8 ) = x 4 14 x 3 + 51 x 2 14 x 80 = 0 (x+1)(x-2)(x-5)(x-8) = x^4 -14x^3 + 51x^2 - 14x-80 = 0 with ( a , b ) = ( 14 , 80 ) a + b = 94 . (a,b) = (-14,-80) \implies a+b= \boxed{-94} .

Tom Engelsman
Dec 6, 2020

Let the four roots be in arithmetic progression: r , r + δ , r + 2 δ , r + 3 δ r, r+ \delta, r + 2\delta, r + 3\delta such that by Vieta's Formulae:

Σ i = 1 4 r i = 4 r + 6 δ = 14 ; \Sigma_{i=1}^{4} r_{i} = 4r+ 6\delta = 14;

Σ i = 1 3 Σ j = i + 1 4 r i r j = 6 r 2 + 18 δ r + 11 δ 2 = 51 \Sigma_{i=1}^{3} \Sigma_{j=i+1}^{4} r_{i}r_{j} = 6r^2 + 18\delta r +11\delta^{2} = 51

which yields ( r , δ ) = ( 1 , 3 ) ; ( 8 , 3 ) (r,\delta) = (-1,3); (8,-3) , and the four roots are 1 , 2 , 5 , 8 -1, 2, 5, 8 . Now we calculate:

a = [ Σ i = 1 2 Σ j = i + 1 3 Σ k = j + 1 4 r i r j r k ] = 14 a = -[\Sigma_{i=1}^{2} \Sigma_{j=i+1}^{3} \Sigma_{k=j+1}^{4} r_{i}r_{j}r_{k} ]= -14 ;

b = Π i = 1 4 r i = 80 b = \Pi_{i=1}^{4} r_{i} = -80

Hence, a + b = 94 . a+b = \boxed{-94}.

0 pending reports

×

Problem Loading...

Note Loading...

Set Loading...